act 9_ quiz 2 robótica

Upload: charryperdomo

Post on 02-Jun-2018

214 views

Category:

Documents


0 download

TRANSCRIPT

  • 8/10/2019 Act 9_ Quiz 2 robtica

    1/8

    18/6/2014 299011-179: Act 9: Quiz 2

    http://www.unad.learnmate.co/mod/quiz/review.php?q=1950&attempt=68236

    ROBOTICAUsted se ha autentificado como MANTILLA, JENIFFER(Salir)

    UNAD ECBTI 299011-179 Cuestionarios Act 9: Quiz 2 Revisin del intento 1

    1

    Puntos:1 Seleccione

    unarespuesta.

    a. Los limites de desplazamiento de sus articulaciones

    b. Los limites de giro de sus articulaciones

    c. La cantidad de eslabones

    d. La exactidtud y precisin de los motores

    La zona de trabajo de un robot manipulador NO se ve restringida por

    CorrectoPuntos para este envo: 1/1.

    2

    Puntos:1 Seleccione

    una

    respuesta.

    a. Evita la utilizacin de coordenadas de diferente tipo para cadaarticulacin.

    b. Se resuelve al encontrar una matriz homognea T querelacione la posicin y orientacin del extremo del robot, respectodel sistema de referencia fijo situado en la base del mismo.

    c. Nunca necesita de clculos matriciales para ser resuelto.

    d. Se resuelve al encontrar una matriz homognea T querelacione la posicin y orientacin del extremo del robot respecto delsistema de referencia fijo situado en la ltima articulacin.

    Escorrecto afirmar que el problema cinemtico directo:

    CorrectoPuntos para este envo: 1/1.

    Act 9: Quiz 2

    Revisin del intento 1

    Finalizar revisin

    Comenzado el viernes, 25 de abril de 2014, 22:03

    Completado el viernes, 25 de abril de 2014, 22:48

    Tiempo empleado 45 minutos 6 segundos

    Puntos 10.67/15

    Calificacin 24.9de un mximo de 35 (71%)

    http://www.unad.learnmate.co/user/view.php?id=5890&course=373http://www.unad.learnmate.co/logout_redirect.phphttp://www.unad.learnmate.co/http://www.unad.learnmate.co/course/view.php?id=373http://www.unad.learnmate.co/mod/quiz/index.php?id=373http://www.unad.learnmate.co/mod/quiz/view.php?id=30803http://www.unad.learnmate.co/mod/quiz/view.php?id=30803http://www.unad.learnmate.co/mod/quiz/index.php?id=373http://www.unad.learnmate.co/course/view.php?id=373http://www.unad.learnmate.co/http://www.unad.learnmate.co/logout_redirect.phphttp://www.unad.learnmate.co/user/view.php?id=5890&course=373
  • 8/10/2019 Act 9_ Quiz 2 robtica

    2/8

    18/6/2014 299011-179: Act 9: Quiz 2

    http://www.unad.learnmate.co/mod/quiz/review.php?q=1950&attempt=68236

    3

    Puntos:1

    Seleccioneuna

    respuesta.

    a. el movimiento con respecto a un sistema de referencia, sinestudiar las causas.

    b. la velocidad y posicin de los actuadores

    c. el movimiento y las causas del mismo con respecto a unsistema de referencia

    d. la velocidad de respuesta de los actuadores

    La cinemtica en el mbito de la robtica estudia:

    IncorrectoPuntos para este envo: 0/1.

    4

    Puntos:1

    Seleccioneunarespuesta.

    a. El robot cuenta con unacmara

    b. El robot cuenta con unsensor de contacto

    Incorrecto, el sensor midedistancias no es de contacto.

    c. El robot cuenta con unsensor para medir distancias

    d. El robot cuenta con unsensor para medir el ngulo

    Contexto: Este tipo de pregunta se desarrolla en torno a un (1) enunciado y cuatro (4)opciones de respuesta. Solo una (1) de estas opciones responde correctamente a lapregunta

    Enunciado: De acuerdo al ejemplo 1 de la leccin 18, se puede afirmar correctamenteque:

    IncorrectoPuntos para este envo: 0/1.

    5

    Puntos:1

    Seleccioneunarespuesta.

    a. Si dos sistemas de referenciaestan relacionados mediante estamatriz de rotacin, entonces susorgenes estan desplazados 0,5unidades en el eje "Y"

    b. Si dos sistemas de referencia

    Usando la ecuacin 2.11 para un ngulo de rotacin de 30 grados, determine cul delas siguientes afirmaciones es verdadera.

  • 8/10/2019 Act 9_ Quiz 2 robtica

    3/8

    18/6/2014 299011-179: Act 9: Quiz 2

    http://www.unad.learnmate.co/mod/quiz/review.php?q=1950&attempt=68236

    estan relacionados mediante estamatriz de rotacin, entonces susorgenes estan desplazados 0,5unidades en el eje "Z"

    c. Si dos sistemas de referenciaestan relacionados mediante estamatriz de rotacin, entonces susorgenes estan desplazados 0,86

    unidades en el eje "X"

    Incorrecto, si lossistemas solo seencuentran rotados unosobre el otro, no tiene

    porque desplazarse elorigen.

    d. Si dos sistemas de referenciaestan relacionados mediante estamatriz de rotacin, entonces susorgenes coinciden

    IncorrectoPuntos para este envo: 0/1.

    6

    Puntos:1

    Seleccioneunarespuesta.

    a. -0.5

    b. 0.86

    c. -0.86

    d. 0.5

    Si el valor del ngulo de rotacin es 30 grados sexagesimales, determine el valor

    numrico de la componente (2,1) de la matriz inversa de .

    CorrectoPuntos para este envo: 1/1.

    7

    Puntos:1

  • 8/10/2019 Act 9_ Quiz 2 robtica

    4/8

    18/6/2014 299011-179: Act 9: Quiz 2

    http://www.unad.learnmate.co/mod/quiz/review.php?q=1950&attempt=68236

    Seleccioneal menosunarespuesta.

    a. El problema planteado es de cinemtica inversa.

    b. Al resolver el problema el resultado es: X=-20 y Y=-20

    c. Al resolver el problema el resultado es: X=0 y Y=0

    d. El problema planteado es de cinemtica directa.

    Teniendo en cuenta los siguientes datos para la figura 1: L1 es la magnitud del eslabn de la base

    y mide 5[cm]. L2 es la magnitud del eslabn restante es de 5[cm], theta_1=0 y theta_2=180

    Determine la posicin del extremo.

    Segn el problema anterior cuales de las siguientes afirmaciones son verdaderas:

    CorrectoPuntos para este envo: 1/1.

    8

    Puntos:1

    Dada la matrizA=

    y dada la matrizB=

    Realice la siguiente operacion P = (A.B)T+ A ,y seleccione el valor numrico de la

  • 8/10/2019 Act 9_ Quiz 2 robtica

    5/8

    18/6/2014 299011-179: Act 9: Quiz 2

    http://www.unad.learnmate.co/mod/quiz/review.php?q=1950&attempt=68236

    Seleccioneunarespuesta.

    a. tres

    b. uno

    c. cero

    d. dos

    componente (3,3) de la matriz resultante P:

    CorrectoPuntos para este envo: 1/1.

    9

    Puntos:1

    Seleccioneunarespuesta.

    a. En un ambiente tridimensional, el tamaode la sub-matriz de traslacin es 3X1

    b. En un ambiente tridimensional, el tamaode la sub-matriz de rotacin es 2X2

    c. En un ambiente tridimensional, el tamaode la sub-matriz de rotacin es 4X4

    Incorrecto,revisar la ecuacin2.22

    d. En un ambiente tridimensional, el tamaode la matriz de perspectiva es 4X4

    Contexto: Este tipo de pregunta se desarrolla en torno a un (1) enunciado y cuatro (4)opciones de respuesta. Solo una (1) de estas opciones responde correctamente a lapregunta

    Enunciado: De acuerdo a lo mencionado en el mdulo seleccione cul de las siguientes

    afirmaciones es verdadera.

    IncorrectoPuntos para este envo: 0/1.

    10

    Puntos:1

    Seleccioneunarespuesta.

    a. Rotacin, radiacin, perspectiva y escalado

    b. Rotacin, mediacin, prospectiva y escalado

    c. Rotacin, traslacin, perspectiva y escalado Muy bien

    d. Rotacin, perspectiva, transfomacin y escalado

    Contexto: Este tipo de pregunta se desarrolla en torno a un (1) enunciado y cuatro (4)opciones de respuesta. Solo una (1) de estas opciones responde correctamente a lapregunta

    Enunciado: De acuerdo a lo mencionado en el mdulo cules son las cuatro sub-matrices de la matriz de transformacin homognea?

  • 8/10/2019 Act 9_ Quiz 2 robtica

    6/8

    18/6/2014 299011-179: Act 9: Quiz 2

    http://www.unad.learnmate.co/mod/quiz/review.php?q=1950&attempt=68236

    CorrectoPuntos para este envo: 1/1.

    11

    Puntos:1

    Seleccioneunarespuesta.

    a. 0.5

    b. 0.86

    c. -0.86

    d. -0.5

    Si el valor del ngulo de rotacin es 30 grados sexagesimales, determine el valor

    numrico de la componente (1,2) de la matriz inversa de .

    CorrectoPuntos para este envo: 1/1.

    12

    Puntos:1

    De acuerdo a la ecuacin 2.36 del mdulo, indique cul de las siguientes operacioneses correcta:

  • 8/10/2019 Act 9_ Quiz 2 robtica

    7/8

    18/6/2014 299011-179: Act 9: Quiz 2

    http://www.unad.learnmate.co/mod/quiz/review.php?q=1950&attempt=68236

    Seleccioneunarespuesta.

    a. opcin 2

    b. opcin 1 Correcto muy bien.

    c. opcin 4

    d. opcin 3

    CorrectoPuntos para este envo: 1/1.

    13

    Puntos:

    1

    Seleccioneunarespuesta.

    a. Un robot articulado puede describirse totalmente usando tresmagnitudes asociadas a cada articulacin

    b. Un robot articulado puede describirse totalmente usando cuatromagnitudes asociadas a cada articulacin.

    c. Un robot articulado puede describirse totalmente, cuando elnmero de articulaciones es par.

    d. Un robot articulado no puede describirse totalmente, cuando el

    Resolver el problema de cinemtica directa requiere de la determinacin de los parmetros de

    Denavit y Hartenberg, quienes afirman que:

  • 8/10/2019 Act 9_ Quiz 2 robtica

    8/8

    18/6/2014 299011-179: Act 9: Quiz 2

    http://www.unad.learnmate.co/mod/quiz/review.php?q=1950&attempt=68236

    nmero de articulaciones es par.

    CorrectoPuntos para este envo: 1/1.

    14

    Puntos:1

    Seleccioneal menosunarespuesta.

    a. el movimiento y las causas del mismo con respecto a unsistema de referencia

    b. la velocidad de respuesta de los actuadores

    c. la velocidad y posicin de los actuadores

    d. el movimiento con respecto a un sistema de referencia, sinestudiar las causas.

    Cuales de las siguientes definiciones no corresponden con el rea de estudio de lacinemtica:

    Parcialmente correctoPuntos para este envo: 0.7/1.

    15

    Puntos:1

    Seleccioneuna

    respuesta.

    a. Utotalmente, cuando el nmero de articulaciones es par

    b. Un robot articulado puede describirse totalmente usando cuatromagnitudes asociadas a cada articulacin.

    c. Un robot articulado no puede describirse totalmente, cuando elnmero de articulaciones es impar.

    d. Un robot articulado puede describirse totalmente usando tresmagnitudes asociadas a cada articulacin

    Denavit y Hartenberg afirman que:

    CorrectoPuntos para este envo: 1/1.

    Finalizar revisin

    Usted se ha autentificado como MANTILLA, JENIFFER(Salir)

    299011-179

    http://www.unad.learnmate.co/course/view.php?id=373http://www.unad.learnmate.co/logout_redirect.phphttp://www.unad.learnmate.co/user/view.php?id=5890&course=373